Fix solution rounding in Serway and Jewett v8's problem 27.1.
[course.git] / latex / problems / Serway_and_Jewett_8 / problem27.01.tex
1 \begin{problem*}{27.1}
2 A proton beam in an accelerator carries a current of $125\U{$\mu$A}$
3 If the beam is incident on a target, how many protons strike the
4 target in a period of $23.0\U{s}$.
5 \end{problem*}
6
7 \begin{solution}
8 Current is defined as $I\equiv\deriv{t}{Q}$.  Therefore, strike rate is
9 \begin{equation}
10   \deriv{t}{N} = \deriv{t}{Q} \cdot \frac{1\U{proton}}{q} = \frac{I}{q} \;,
11 \end{equation}
12 where $q=1.60\E{-19}\U{C}$ is the charge of a single proton.  The
13 number of strikes in the allotted time is thus
14 \begin{equation}
15   \Delta N = \deriv{t}{N} \cdot \Delta t = \frac{I\Delta t}{q}
16     = \ans{18.0\E{15}\U{protons} = 18.0\U{petaprotons}} \;.
17 \end{equation}
18 \end{solution}